Question:

The incorrect statement regarding the given circuit
potential at A is 10 V

Show Hint

Analyze the potential differences across each element in the circuit based on the given options. Use Kirchhoff's Voltage Law around closed loops and Kirchhoff's Current Law at junctions to check for consistency. Look for contradictions in the potential values implied by different parts of the circuit. The statement that leads to an inconsistency is the incorrect statement.
Updated On: May 12, 2025
  • current through 2 \( \Omega \) is 2A
  • current through 3 \( \Omega \) is 4A
  • potential at C is 12 V
  • potential at A is 10 V
Hide Solution
collegedunia
Verified By Collegedunia

The Correct Option is D

Solution and Explanation

Let's analyze the circuit using Kirchhoff's laws.
Assume the potential at point C is \( V_C \).
Applying Kirchhoff's Voltage Law (KVL) to the loop containing the 6V battery, 2 \( \Omega \) resistor, and the path from A to C: \( 6 - 2 I_1 - (V_A - V_C) = 0 \) (Equation 1) where \( I_1 \) is the current through the 2 \( \Omega \) resistor, flowing from C to A.
Applying KVL to the loop containing the 2V battery, 3 \( \Omega \) resistor, and the path from D to C: \( 2 - 3 I_2 - (V_D - V_C) = 0 \) (Equation 2) where \( I_2 \) is the current through the 3 \( \Omega \) resistor, flowing from C to D.
Applying KVL to the loop containing the 10V battery, the path from A to B, and the path from B to D: \( 10 - (V_A - V_B) - (V_B - V_D) = 0 \) \( 10 - V_A + V_D = 0 \) \( V_A - V_D = 10 \) (Equation 3) Let's assume the potential at point C is 0 V for simplicity.
Then the potential at the positive terminal of the 6V battery is 6V higher than C, and the potential at the positive terminal of the 2V battery is 2V higher than C.
Consider the currents at junction B.
Let the current through the 10V battery be \( I \), flowing from A to B.
Then \( I = I_1 + I_2 \).
Applying KVL to the loop containing the 6V battery, 2 \( \Omega \) resistor, 10V battery, and 3 \( \Omega \) resistor: \( 6 - 2 I_1 + 10 - 3 I_2 - 2 = 0 \) \( 14 = 2 I_1 + 3 I_2 \) (Equation 4) We need more information or a different approach to directly find the potentials and currents without assuming a current direction through the 10V battery.
Let's try assuming the potential at C is \( V_C \).
Potential at the positive terminal of the 6V battery is \( V_C + 6 \), which is \( V_A \).
So \( V_A = V_C + 6 \).
Potential at the positive terminal of the 2V battery is \( V_C + 2 \), which is \( V_D \).
So \( V_D = V_C + 2 \).
Substitute \( V_A \) and \( V_D \) in Equation 3: \( (V_C + 6) - (V_C + 2) = 10 \) \( 4 = 10 \) This indicates an inconsistency in the assumed polarities or the circuit diagram.
Let's re-examine the connections.
Assuming the standard interpretation of the circuit diagram: Potential at A is 6V higher than C: \( V_A = V_C + 6 \) Potential at D is 2V higher than C: \( V_D = V_C + 2 \) Potential at B is 10V higher than A: \( V_B = V_A + 10 = V_C + 16 \) Potential at B is also 2V higher than D: \( V_B = V_D + 2 = V_C + 4 \) From \( V_C + 16 = V_C + 4 \), we get \( 16 = 4 \), which is a contradiction.
This implies there is no consistent set of potentials satisfying the given circuit.
There must be a current flowing in the central branch.
Let's use nodal analysis.
Let the potential at C be 0V.
Then \( V_A = 6V \), \( V_D = 2V \).
Let the potential at B be \( V_B \).
Current through 2 \( \Omega \) resistor: \( I_{CA} = \frac{V_A - V_C}{2} = \frac{6 - 0}{2} = 3 \) A (from A to C) Current through 3 \( \Omega \) resistor: \( I_{CD} = \frac{V_D - V_C}{3} = \frac{2 - 0}{3} \) A (from D to C) Current through the 10V branch: \( I_{AB} = \frac{V_A - V_B}{0} \) (resistance is not given) Let's assume the given options are to be checked for consistency.

Option (D) Potential at A is 10 V.
If \( V_A = 10 \) V and \( V_C = 0 \) V, current through 2 \( \Omega \) is \( (10-0)/2 = 5 \) A.
If \( V_A = 10 \) V, then \( V_B = 10 + 10 = 20 \) V.
If \( V_C = 0 \) V, then \( V_D = 2 \) V.
Potential difference across the 3 \( \Omega \) is \( 2 - 0 = 2 \) V, current is \( 2/3 \) A.
Let's assume current through 2 \( \Omega \) is 2A (Option A).
Then \( V_A - V_C = 4 \).
If \( V_C = 0 \), \( V_A = 4 \).
Then \( V_B = 14 \), \( V_D = 2 \).
Current through 3 \( \Omega \) is \( (2-0)/3 \) A.
Let's assume current through 3 \( \Omega \) is 4A (Option B).
Then \( V_D - V_C = 12 \).
If \( V_C = 0 \), \( V_D = 12 \).
Then \( V_B = 14 \), \( V_A = 4 \).
Current through 2 \( \Omega \) is \( (4-0)/2 = 2 \) A.
Let's assume potential at C is 12 V (Option C).
Then \( V_A = 18 \), \( V_D = 14 \), \( V_B = 28 \).
Current through 2 \( \Omega \) is \( (18-12)/2 = 3 \) A, current through 3 \( \Omega \) is \( (14-12)/3 = 2/3 \) A.
If potential at A is 10V, and potential at C is 0V, current through 2\( \Omega \) is 5A.
Potential at B is 20V.
Potential at D is 2V.
Current through 3\( \Omega \) is 2/3A.
This scenario doesn't seem consistent with other parts of the circuit without a current in the 10V branch.
The question asks for the incorrect statement.
Option (D) states potential at A is 10 V.
If we consider a loop from the 6V battery to point A, the potential at A relative to C depends on the current through the 2\( \Omega \) resistor.
Without knowing this current, we cannot definitively say \( V_A = 10 \) V is incorrect.
However, let's re-examine the implications if \( V_A = 10 \) V.
If \( V_C = 0 \), current through 2\( \Omega \) is 5A.
\( V_B = 20 \) V, \( V_D = 2 \) V, current through 3\( \Omega \) is 2/3A.
This doesn't lead to any immediate contradiction without analyzing the central branch.
Let's consider option (A): current through 2 \( \Omega \) is 2A.
If \( I_1 = 2 \) A, \( V_A - V_C = 4 \).
If \( V_C = 0 \), \( V_A = 4 \).
Then \( V_B = 14 \), \( V_D = 2 \).
Current through 3 \( \Omega \) is \( (2-0)/3 \) A.
This doesn't immediately show inconsistency.
Let's consider option (B): current through 3 \( \Omega \) is 4A.
If \( I_2 = 4 \) A, \( V_D - V_C = 12 \).
If \( V_C = 0 \), \( V_D = 12 \).
Then \( V_B = 14 \), \( V_A = 4 \).
Current through 2 \( \Omega \) is \( (4-0)/2 = 2 \) A.
This seems consistent so far.
Let's consider option (C): potential at C is 12 V.
Then \( V_A = 18 \), \( V_D = 14 \), \( V_B = 28 \).
Currents are consistent.
Let's revisit option (D): potential at A is 10 V.
If \( V_C = 0 \), \( V_A = 10 \).
Then \( V_B = 20 \), \( V_D = 2 \).
The potential difference across the 10V battery is \( V_B - V_A = 10 \) V, which is consistent with its EMF.
However, without knowing the resistance in the central branch, we cannot confirm the currents.
Given the inconsistency found earlier with potential differences around the loop containing the batteries, there might be an issue with the circuit diagram itself or the intended interpretation.
However, based on the options, let's try to find a contradiction.
If \( V_A = 10 \) V and \( V_C = 0 \) V, current through 2 \( \Omega \) is 5A.
\( V_B = 20 \) V, \( V_D = 2 \) V, current through 3 \( \Omega \) is 2/3A.
Applying KCL at B: current from A to B should equal current from B to D.
\( \frac{V_B - V_A}{0} \) which is undefined unless \( V_B = V_A \), contradicting \( V_B = V_A + 10 \).
Thus, \( V_A = 10 \) V leads to a contradiction.
Final Answer: The final answer is $\boxed{potential at A is 10 V}$
Was this answer helpful?
0
0